Difference between revisions of "2012 AMC 8 Problems/Problem 7"

m (Solution 2)
(Solution 2)
Line 8: Line 8:
  
 
==Solution 2==
 
==Solution 2==
Isabella needs to lose a max of <math>20</math>. When she got <math>97</math> and <math>91</math>, the max goes down to <math>20 - 3 - 9 = 8</math>. If we get <math>100</math> on her last test, then she will get <math>\boxed{\textbf{(B)}\ 92}</math> on the thrid test.
+
Isabella needs to lose a max of <math>20</math>. When she got <math>97</math> and <math>91</math>, the max goes down to <math>20 - 3 - 9 = 8</math>. If we get <math>100</math> on her last test, then she will get <math>\boxed{\textbf{(B)}\ 92}</math> on the third test.
  
 
==See Also==
 
==See Also==
 
{{AMC8 box|year=2012|num-b=6|num-a=8}}
 
{{AMC8 box|year=2012|num-b=6|num-a=8}}
 
{{MAA Notice}}
 
{{MAA Notice}}

Revision as of 19:32, 1 December 2020

Problem

Isabella must take four 100-point tests in her math class. Her goal is to achieve an average grade of 95 on the tests. Her first two test scores were 97 and 91. After seeing her score on the third test, she realized she can still reach her goal. What is the lowest possible score she could have made on the third test?

$\textbf{(A)}\hspace{.05in}90\qquad\textbf{(B)}\hspace{.05in}92\qquad\textbf{(C)}\hspace{.05in}95\qquad\textbf{(D)}\hspace{.05in}96\qquad\textbf{(E)}\hspace{.05in}97$

Solution 1

Isabella wants an average grade of $95$ on her 4 tests; this also means that she wants the sum of her test scores to be at least $95 \times 4 = 380$ (if she goes over this number, she'll be over her goal!). She's already taken two tests, which sum to $97+91 = 188$, which means she needs $192$ more points to achieve her desired average. In order to minimize the score on the third test, we assume that Isabella will receive all $100$ points on the fourth test. Therefore, the lowest Isabella could have scored on the third test would be $192-100 = \boxed{\textbf{(B)}\ 92}$.

Solution 2

Isabella needs to lose a max of $20$. When she got $97$ and $91$, the max goes down to $20 - 3 - 9 = 8$. If we get $100$ on her last test, then she will get $\boxed{\textbf{(B)}\ 92}$ on the third test.

See Also

2012 AMC 8 (ProblemsAnswer KeyResources)
Preceded by
Problem 6
Followed by
Problem 8
1 2 3 4 5 6 7 8 9 10 11 12 13 14 15 16 17 18 19 20 21 22 23 24 25
All AJHSME/AMC 8 Problems and Solutions

The problems on this page are copyrighted by the Mathematical Association of America's American Mathematics Competitions. AMC logo.png